Proving roots using mean value theorem

Click For Summary
To prove that the equation x^4 + 4x + c = 0 has at most two real roots, the discussion centers on assuming the existence of three real roots and seeking a contradiction. By applying the Mean Value Theorem (MVT), it is established that if there are three roots, there must be at least three points where the derivative f'(x) = 0. However, the derivative f'(x) = 4x^3 + 4 has only one stationary point at x = -1, indicating that there cannot be three distinct roots. This contradiction confirms that the original assumption is false, thereby proving that the polynomial can have at most two real roots. The value of c does not affect this conclusion.
Jeff Ford
Messages
154
Reaction score
2
Prove x^4 + 4x + c = 0 has at most two real roots

My thinking is that to prove this I would assume that it has three real roots and look for a contradiction.

So I set f(x) = x^4 + 4x + c and assume three real roots x_1, x_2, x_3 such that f(x_1) = f(x_2) = f(x_3) = 0

By MVT I know that there must exist c_1 on the interval (x_1, x_2) such that f'(c_1) = 0 and c_2 on the interval (x_2, x_3) such that f'(c_2) = 0 and c_3 on the interval (x_1, x_3) such that f'(c_3) = 0

Now I'm a little confused. Do I try to find three values of c that will satisfy f'(x) = 4x^3 + 4 = 0? Only the value x = -1 would make this true, so is that my contradiction? That MVT predicts three values and only one exists?

Thanks
Jeff
 
Last edited:
Physics news on Phys.org
Hint:
How many STATIONARY points does the function have?
 
Only one, at (-1, 5+c). Since the derivative is negative to the left of that point and positive to the right, it must be a minimum of the function.
 
Last edited:
I assume you orden x1 < x2 < x3. It could be the case that f'(c3) equals f'(c1) or f'(c2), so according to the MVT you'll get at least 2 points where f' is zero. You've shown there's only one, so the assumption of three distinct roots is false. Not sure why you want to find particular values of c to make f' zero. The value of c is irrelevant to the problem.
 
Thanks for the clarification
 
Question: A clock's minute hand has length 4 and its hour hand has length 3. What is the distance between the tips at the moment when it is increasing most rapidly?(Putnam Exam Question) Answer: Making assumption that both the hands moves at constant angular velocities, the answer is ## \sqrt{7} .## But don't you think this assumption is somewhat doubtful and wrong?

Similar threads

  • · Replies 2 ·
Replies
2
Views
1K
Replies
14
Views
2K
  • · Replies 9 ·
Replies
9
Views
1K
  • · Replies 11 ·
Replies
11
Views
2K
  • · Replies 3 ·
Replies
3
Views
900
  • · Replies 7 ·
Replies
7
Views
3K
  • · Replies 2 ·
Replies
2
Views
3K
  • · Replies 26 ·
Replies
26
Views
3K
  • · Replies 1 ·
Replies
1
Views
1K
  • · Replies 9 ·
Replies
9
Views
2K